Difference between revisions of "1997 AJHSME Problems/Problem 14"

(Solution)
Line 7: Line 7:
 
==Solution==
 
==Solution==
  
Call the set <math>\{a, b, c, d, e\}</math>, with <math>a \le b \le c \le d \le e</math>.
+
JO MAMA
 
 
Since the median, or middle number, is <math>5</math>, we have <math>c=5</math>.
 
 
 
Since the mode, or most common number, is <math>8</math>, we have <math>d=e=8</math>.
 
 
 
Thus, the set is now <math>\{a, b, 5, 8, 8\}</math>.  Additionally, since there is only one mode, <math>a</math> and <math>b</math> are both less than <math>5</math> and are distinct from each other.
 
 
 
Since the mean of the set is <math>5</math>, and there are five numbers in the set, the sum of the numbers in the set is <math>5\times 5 = 25</math>.
 
 
 
Thus, we have <math>a + b + 5 + 8 + 8 = 25</math>, which leads to <math>a + b = 4</math>
 
 
 
Since <math>a</math> and <math>b</math> are distinct positive integers, they must equal <math>1</math> and <math>3</math>.
 
 
 
The smallest number is <math>1</math>, and the largest number is <math>8</math>, giving a difference of <math>7</math>, leading to answer <math>\boxed{D}</math>
 
  
 
== See also ==
 
== See also ==

Revision as of 18:51, 16 April 2019

Problem

There is a set of five positive integers whose average (mean) is 5, whose median is 5, and whose only mode is 8. What is the difference between the largest and smallest integers in the set?

$\text{(A)}\ 3 \qquad \text{(B)}\ 5 \qquad \text{(C)}\ 6 \qquad \text{(D)}\ 7 \qquad \text{(E)}\ 8$

Solution

JO MAMA

See also

1997 AJHSME (ProblemsAnswer KeyResources)
Preceded by
Problem 13
Followed by
Problem 15
1 2 3 4 5 6 7 8 9 10 11 12 13 14 15 16 17 18 19 20 21 22 23 24 25
All AJHSME/AMC 8 Problems and Solutions

The problems on this page are copyrighted by the Mathematical Association of America's American Mathematics Competitions. AMC logo.png